Find integer solutions for $m^3-n^2=2$












1












$begingroup$



Find integer solutions for $m^3-n^2=2$




I have seen multiple solutions like this one.



However, these have been complicated. I tried taking different modulos but could not achieve anything. My latest attempt included this:
$$m^3+1=n^2-1$$
$$(m+1)(m^2-m+1)=(n-1)(n+1)$$
But this does not help. Is there any solution that is easy. Thank you in advance.










share|cite|improve this question











$endgroup$








  • 1




    $begingroup$
    For what it's worth, the equation $$y^2 = x^3 - 2$$ defines an elliptic curve, with the two lattice points $(3, 5)$ and $(3, -5)$.
    $endgroup$
    – Jose Arnaldo Bebita Dris
    Dec 23 '18 at 7:46








  • 2




    $begingroup$
    BTW, the equations $$m^3 - n^2 = 2$$ and $$m^3 + 1 = n^2 - 1$$ do not have the same solutions.
    $endgroup$
    – Jose Arnaldo Bebita Dris
    Dec 23 '18 at 7:51






  • 2




    $begingroup$
    The method in the linked thread is from several textbooks on number theory. The reason is probably that it is the simplest solution known to mankind. Modular arithmetic is unlikely to do much good exactly because we have that solution $m=3,n=5$. With respect to any modulus there will be solutions congruent to that. Some kind of a descent might still be possible (compare to the standard proof of irrationality of $sqrt2$), but if one existed, surely the number theory books would use that instead.
    $endgroup$
    – Jyrki Lahtonen
    Dec 23 '18 at 7:52










  • $begingroup$
    You might ask to simply explain that solution instead.
    $endgroup$
    – tarit goswami
    Dec 23 '18 at 7:56






  • 3




    $begingroup$
    @taritgoswami: I refer you to this MathOverflow Q&A, and I quote: "Finding all the integral points on an elliptic curve is a non-trivial computational problem."
    $endgroup$
    – Jose Arnaldo Bebita Dris
    Dec 23 '18 at 8:05
















1












$begingroup$



Find integer solutions for $m^3-n^2=2$




I have seen multiple solutions like this one.



However, these have been complicated. I tried taking different modulos but could not achieve anything. My latest attempt included this:
$$m^3+1=n^2-1$$
$$(m+1)(m^2-m+1)=(n-1)(n+1)$$
But this does not help. Is there any solution that is easy. Thank you in advance.










share|cite|improve this question











$endgroup$








  • 1




    $begingroup$
    For what it's worth, the equation $$y^2 = x^3 - 2$$ defines an elliptic curve, with the two lattice points $(3, 5)$ and $(3, -5)$.
    $endgroup$
    – Jose Arnaldo Bebita Dris
    Dec 23 '18 at 7:46








  • 2




    $begingroup$
    BTW, the equations $$m^3 - n^2 = 2$$ and $$m^3 + 1 = n^2 - 1$$ do not have the same solutions.
    $endgroup$
    – Jose Arnaldo Bebita Dris
    Dec 23 '18 at 7:51






  • 2




    $begingroup$
    The method in the linked thread is from several textbooks on number theory. The reason is probably that it is the simplest solution known to mankind. Modular arithmetic is unlikely to do much good exactly because we have that solution $m=3,n=5$. With respect to any modulus there will be solutions congruent to that. Some kind of a descent might still be possible (compare to the standard proof of irrationality of $sqrt2$), but if one existed, surely the number theory books would use that instead.
    $endgroup$
    – Jyrki Lahtonen
    Dec 23 '18 at 7:52










  • $begingroup$
    You might ask to simply explain that solution instead.
    $endgroup$
    – tarit goswami
    Dec 23 '18 at 7:56






  • 3




    $begingroup$
    @taritgoswami: I refer you to this MathOverflow Q&A, and I quote: "Finding all the integral points on an elliptic curve is a non-trivial computational problem."
    $endgroup$
    – Jose Arnaldo Bebita Dris
    Dec 23 '18 at 8:05














1












1








1


1



$begingroup$



Find integer solutions for $m^3-n^2=2$




I have seen multiple solutions like this one.



However, these have been complicated. I tried taking different modulos but could not achieve anything. My latest attempt included this:
$$m^3+1=n^2-1$$
$$(m+1)(m^2-m+1)=(n-1)(n+1)$$
But this does not help. Is there any solution that is easy. Thank you in advance.










share|cite|improve this question











$endgroup$





Find integer solutions for $m^3-n^2=2$




I have seen multiple solutions like this one.



However, these have been complicated. I tried taking different modulos but could not achieve anything. My latest attempt included this:
$$m^3+1=n^2-1$$
$$(m+1)(m^2-m+1)=(n-1)(n+1)$$
But this does not help. Is there any solution that is easy. Thank you in advance.







elementary-number-theory






share|cite|improve this question















share|cite|improve this question













share|cite|improve this question




share|cite|improve this question








edited Dec 23 '18 at 7:48







user587054

















asked Dec 23 '18 at 7:42









user587054user587054

56511




56511








  • 1




    $begingroup$
    For what it's worth, the equation $$y^2 = x^3 - 2$$ defines an elliptic curve, with the two lattice points $(3, 5)$ and $(3, -5)$.
    $endgroup$
    – Jose Arnaldo Bebita Dris
    Dec 23 '18 at 7:46








  • 2




    $begingroup$
    BTW, the equations $$m^3 - n^2 = 2$$ and $$m^3 + 1 = n^2 - 1$$ do not have the same solutions.
    $endgroup$
    – Jose Arnaldo Bebita Dris
    Dec 23 '18 at 7:51






  • 2




    $begingroup$
    The method in the linked thread is from several textbooks on number theory. The reason is probably that it is the simplest solution known to mankind. Modular arithmetic is unlikely to do much good exactly because we have that solution $m=3,n=5$. With respect to any modulus there will be solutions congruent to that. Some kind of a descent might still be possible (compare to the standard proof of irrationality of $sqrt2$), but if one existed, surely the number theory books would use that instead.
    $endgroup$
    – Jyrki Lahtonen
    Dec 23 '18 at 7:52










  • $begingroup$
    You might ask to simply explain that solution instead.
    $endgroup$
    – tarit goswami
    Dec 23 '18 at 7:56






  • 3




    $begingroup$
    @taritgoswami: I refer you to this MathOverflow Q&A, and I quote: "Finding all the integral points on an elliptic curve is a non-trivial computational problem."
    $endgroup$
    – Jose Arnaldo Bebita Dris
    Dec 23 '18 at 8:05














  • 1




    $begingroup$
    For what it's worth, the equation $$y^2 = x^3 - 2$$ defines an elliptic curve, with the two lattice points $(3, 5)$ and $(3, -5)$.
    $endgroup$
    – Jose Arnaldo Bebita Dris
    Dec 23 '18 at 7:46








  • 2




    $begingroup$
    BTW, the equations $$m^3 - n^2 = 2$$ and $$m^3 + 1 = n^2 - 1$$ do not have the same solutions.
    $endgroup$
    – Jose Arnaldo Bebita Dris
    Dec 23 '18 at 7:51






  • 2




    $begingroup$
    The method in the linked thread is from several textbooks on number theory. The reason is probably that it is the simplest solution known to mankind. Modular arithmetic is unlikely to do much good exactly because we have that solution $m=3,n=5$. With respect to any modulus there will be solutions congruent to that. Some kind of a descent might still be possible (compare to the standard proof of irrationality of $sqrt2$), but if one existed, surely the number theory books would use that instead.
    $endgroup$
    – Jyrki Lahtonen
    Dec 23 '18 at 7:52










  • $begingroup$
    You might ask to simply explain that solution instead.
    $endgroup$
    – tarit goswami
    Dec 23 '18 at 7:56






  • 3




    $begingroup$
    @taritgoswami: I refer you to this MathOverflow Q&A, and I quote: "Finding all the integral points on an elliptic curve is a non-trivial computational problem."
    $endgroup$
    – Jose Arnaldo Bebita Dris
    Dec 23 '18 at 8:05








1




1




$begingroup$
For what it's worth, the equation $$y^2 = x^3 - 2$$ defines an elliptic curve, with the two lattice points $(3, 5)$ and $(3, -5)$.
$endgroup$
– Jose Arnaldo Bebita Dris
Dec 23 '18 at 7:46






$begingroup$
For what it's worth, the equation $$y^2 = x^3 - 2$$ defines an elliptic curve, with the two lattice points $(3, 5)$ and $(3, -5)$.
$endgroup$
– Jose Arnaldo Bebita Dris
Dec 23 '18 at 7:46






2




2




$begingroup$
BTW, the equations $$m^3 - n^2 = 2$$ and $$m^3 + 1 = n^2 - 1$$ do not have the same solutions.
$endgroup$
– Jose Arnaldo Bebita Dris
Dec 23 '18 at 7:51




$begingroup$
BTW, the equations $$m^3 - n^2 = 2$$ and $$m^3 + 1 = n^2 - 1$$ do not have the same solutions.
$endgroup$
– Jose Arnaldo Bebita Dris
Dec 23 '18 at 7:51




2




2




$begingroup$
The method in the linked thread is from several textbooks on number theory. The reason is probably that it is the simplest solution known to mankind. Modular arithmetic is unlikely to do much good exactly because we have that solution $m=3,n=5$. With respect to any modulus there will be solutions congruent to that. Some kind of a descent might still be possible (compare to the standard proof of irrationality of $sqrt2$), but if one existed, surely the number theory books would use that instead.
$endgroup$
– Jyrki Lahtonen
Dec 23 '18 at 7:52




$begingroup$
The method in the linked thread is from several textbooks on number theory. The reason is probably that it is the simplest solution known to mankind. Modular arithmetic is unlikely to do much good exactly because we have that solution $m=3,n=5$. With respect to any modulus there will be solutions congruent to that. Some kind of a descent might still be possible (compare to the standard proof of irrationality of $sqrt2$), but if one existed, surely the number theory books would use that instead.
$endgroup$
– Jyrki Lahtonen
Dec 23 '18 at 7:52












$begingroup$
You might ask to simply explain that solution instead.
$endgroup$
– tarit goswami
Dec 23 '18 at 7:56




$begingroup$
You might ask to simply explain that solution instead.
$endgroup$
– tarit goswami
Dec 23 '18 at 7:56




3




3




$begingroup$
@taritgoswami: I refer you to this MathOverflow Q&A, and I quote: "Finding all the integral points on an elliptic curve is a non-trivial computational problem."
$endgroup$
– Jose Arnaldo Bebita Dris
Dec 23 '18 at 8:05




$begingroup$
@taritgoswami: I refer you to this MathOverflow Q&A, and I quote: "Finding all the integral points on an elliptic curve is a non-trivial computational problem."
$endgroup$
– Jose Arnaldo Bebita Dris
Dec 23 '18 at 8:05










0






active

oldest

votes











Your Answer





StackExchange.ifUsing("editor", function () {
return StackExchange.using("mathjaxEditing", function () {
StackExchange.MarkdownEditor.creationCallbacks.add(function (editor, postfix) {
StackExchange.mathjaxEditing.prepareWmdForMathJax(editor, postfix, [["$", "$"], ["\\(","\\)"]]);
});
});
}, "mathjax-editing");

StackExchange.ready(function() {
var channelOptions = {
tags: "".split(" "),
id: "69"
};
initTagRenderer("".split(" "), "".split(" "), channelOptions);

StackExchange.using("externalEditor", function() {
// Have to fire editor after snippets, if snippets enabled
if (StackExchange.settings.snippets.snippetsEnabled) {
StackExchange.using("snippets", function() {
createEditor();
});
}
else {
createEditor();
}
});

function createEditor() {
StackExchange.prepareEditor({
heartbeatType: 'answer',
autoActivateHeartbeat: false,
convertImagesToLinks: true,
noModals: true,
showLowRepImageUploadWarning: true,
reputationToPostImages: 10,
bindNavPrevention: true,
postfix: "",
imageUploader: {
brandingHtml: "Powered by u003ca class="icon-imgur-white" href="https://imgur.com/"u003eu003c/au003e",
contentPolicyHtml: "User contributions licensed under u003ca href="https://creativecommons.org/licenses/by-sa/3.0/"u003ecc by-sa 3.0 with attribution requiredu003c/au003e u003ca href="https://stackoverflow.com/legal/content-policy"u003e(content policy)u003c/au003e",
allowUrls: true
},
noCode: true, onDemand: true,
discardSelector: ".discard-answer"
,immediatelyShowMarkdownHelp:true
});


}
});














draft saved

draft discarded


















StackExchange.ready(
function () {
StackExchange.openid.initPostLogin('.new-post-login', 'https%3a%2f%2fmath.stackexchange.com%2fquestions%2f3050142%2ffind-integer-solutions-for-m3-n2-2%23new-answer', 'question_page');
}
);

Post as a guest















Required, but never shown

























0






active

oldest

votes








0






active

oldest

votes









active

oldest

votes






active

oldest

votes
















draft saved

draft discarded




















































Thanks for contributing an answer to Mathematics Stack Exchange!


  • Please be sure to answer the question. Provide details and share your research!

But avoid



  • Asking for help, clarification, or responding to other answers.

  • Making statements based on opinion; back them up with references or personal experience.


Use MathJax to format equations. MathJax reference.


To learn more, see our tips on writing great answers.




draft saved


draft discarded














StackExchange.ready(
function () {
StackExchange.openid.initPostLogin('.new-post-login', 'https%3a%2f%2fmath.stackexchange.com%2fquestions%2f3050142%2ffind-integer-solutions-for-m3-n2-2%23new-answer', 'question_page');
}
);

Post as a guest















Required, but never shown





















































Required, but never shown














Required, but never shown












Required, but never shown







Required, but never shown

































Required, but never shown














Required, but never shown












Required, but never shown







Required, but never shown







Popular posts from this blog

How do I know what Microsoft account the skydrive app is syncing to?

When does type information flow backwards in C++?

Grease: Live!